GR9277 #53
|
|
Problem
|
|
\prob{53}
A particle of mass m is confined to an infinitely deep square-well potential:
The normalized eigenfunctions, labeled by the quantum number n, are
A measurement of energy E will always satisfy which of the following relationships?
-
-
-
-
-
|
Quantum Mechanics}Energy
If one forgets the energy of an infinite well, one can quickly derive it from the time-independent Schrodinger's Equation . However, since inside, one has .
Plug in the ground-state wave function , where . Chunk out the second derivative to get . Plug in k to get .
Note that can be deduced from boundary conditions, i.e., the wave function vanishes at both ends ( and ). The second boundary condition forces the n's to be integers. Since one can't have a trivial wave function, , and thus . One finds that , since , as in choice (E).
|
|
Alternate Solutions |
panos85 2007-10-30 11:31:21 | The energy depends on n so it cannot be a constant. This eliminates C, D, E. Furthermore it cannot be bounded from above as it increases with n. (You can easily remember that.) So A is out. Choice B remains. | |
|
Comments |
francesco 2017-09-14 20:26:58 | if you look at the eigenfunctions (in particular, at the argument of the sine) you can deduce that , plug it in and find choice B | | syreen 2013-09-18 13:47:54 | The Schrodinger EQ should have hbar^2, not just hbar.
Also, as others have noted, the typed question has a typo. B. should have E>=, NOT E<= | | Kabuto Yakushi 2010-09-04 08:58:08 | As long as you remember (which it is probably a good idea to memorize for this test) the energy in a potential well to be
E =
with a note that using results in a two not an eight on the bottom, choice (B) is obvious. (C),(D), and (E) are to precise to satisfy the "always" criteria. | | Fizzics 2009-10-30 11:44:19 | Well I tried to post that last thing as a typo. | | Fizzics 2009-10-30 11:43:33 | The Answer should be "B" not "E". And the answer shown in the problem as "B" is wrong. Should be E is GREATER THAN or EQUAL TO.
Typos. | | jw111 2008-11-04 11:45:30 | Since
and where H = h bar
so
with the solution given by question,
thus
| | Monk 2008-10-13 21:46:07 | (hbar)^2 is missing in the original presentation of the TISE, it just shows (hbar)...though most of you should know that anyhow :P | | adenos 2007-11-02 12:49:53 | B should read E >= ....
evanb 2008-06-23 19:07:46 |
yes.
|
evanb 2008-06-23 19:11:02 |
Dammit, the preview de-TYPO-ified my entry.
Yes, (B) should be >= and not <=
and it should be the answer, not (E)
|
| | panos85 2007-10-30 11:31:21 | The energy depends on n so it cannot be a constant. This eliminates C, D, E. Furthermore it cannot be bounded from above as it increases with n. (You can easily remember that.) So A is out. Choice B remains.
hoyas08 2008-06-16 19:52:47 |
Unfortunately, choice D has an term in it, so you can't eliminate it by reasoning that the energy must depend on n. However, if you can remember that the energy of a particle in free space (V=0) is E=, then the 8 in the denominator of D does not make sense.
|
hoyas08 2008-06-16 19:55:05 |
Unfortunately, choice D has an term in it, so you can't eliminate it by reasoning that the energy must depend on . However, if you can remember that the energy of a particle in free space (V=0) is E=, then the 8 in the denominator of D doesn't make sense, eliminating that answer.
|
Imperate 2008-10-16 10:18:38 |
This is not true for D, D has E=n^2... which is the correct form.rnrnOne can throw away A becayse there is no upper bound to energy.rnOne can throw away C and E because they [b] are [/b] constants, which is unacceptable for the energy.rnrnThis leaves B and D, and D can only be eliminated by working out the correct coeff, as others have posted.rnrnI think this question is a bit harsh though because from elementary QM infinite square wells always have n^2 dependence so you see D whilst doing a high speed test like the GRE then you are going to pick D, well I did anyway....rnrnI
|
ramparts 2009-08-06 19:24:55 |
How does that eliminate D? D is dependent on n.
|
ramparts 2009-08-06 19:24:55 |
How does that eliminate D? D is dependent on n.
|
| | antithesis 2007-10-05 18:17:17 | Aside from typos mentioned, the question itself has a typo, B should have equal or greater than | | Mexicana 2007-10-04 17:46:32 | It is much easier to derive this by getting the zero point energy via Heisenberg Uncertainty Principle and then square this expression. Now just substitute for where is the width of the square well and then the usual expression for momentum .
Tommy Koulax 2007-10-29 09:53:59 |
doesn't that lead to E= which is not an option provided?
|
| | georgi 2007-08-26 20:20:13 | as everyone is posting the answer should be choice b, not e as written | | welshmj 2007-07-12 15:51:47 | Choice B is the correct answer not E, but choice B on this page has a typo. It should be greater than or equal to, not less than or equal to. | | JB 2005-12-07 02:32:15 | Isn't your answer choice (B)
jcain6 2005-12-08 19:20:24 |
It should be choice (B) because the question requires E to ALWAYS satisfy the relationship. Choice (E) only works when the potential is 0 between o and a.
|
Andresito 2006-03-27 15:43:35 |
I also think it is (B).
|
sirius 2008-06-25 13:05:18 |
the potential is 0 from 0 to a. it says so in the problem. this is a particle in an infinite square well, and answer (E) is the only answer that makes sense.
|
sirius 2008-06-25 13:09:59 |
nevermind, i misread which had the >=, i agree it is (B)
|
| | jcain6 2005-11-22 19:06:10 | Your missing a (hbar)^2 in your numerator.
yosun 2005-11-23 02:00:51 |
jcain6: thanks for the typo-alert; it's been corrected.
|
| |
|
|
|
|
The Sidebar Chatbox...
Scroll to see it, or resize your browser to ignore it... |
|
|